1
$\lim_{x \to0}  \left(\dfrac{1}{\sin^2 x}-\dfrac{1}{x^2}\right)$ limitini bulunuz.


2
$a,b,c\in\mathbb{R}$ olmak üzere $a^2+b^2+c^2=a^3+b^3+c^3=1$ ise $a+b+c$ kaçtır?

Tübitak Matematik Olimpiyadı 2022 yılı  1.Aşamada iki boyutlusu sorulmuş: https://geomania.org/forum/index.php?topic=7489.0


3
Her $n$ pozitif tam sayısı ve $a_1, a_2, \ldots, a_n$ pozitif reelleri için

$$\sum_{i=1}^{n} \dfrac{1}{2^i}\left(\dfrac{2}{1+a_i}\right)^{2^i} \geq \dfrac{2}{1+a_1a_2\ldots a_n}-\dfrac{1}{2^n}$$

eşitsizliğinin çalıştığını gösteriniz.


4
Altın oran $\phi=\dfrac{1+\sqrt 5}{2}$  olmak üzere $$\pi\lt2\cdot \phi$$ olduğunu gösteriniz.


5
Her $x,y$ reel sayıları için

$$f(f(y))+f(x-y)=f(xf(y)-x)$$

koşulunu sağlayan tüm $f: \mathbb{R}\to\mathbb{R}$ tanımlanmış $f$ fonksiyonlarını bulunuz.


6
\[\lim_{n\to\infty}\left(n\left(1+\frac1n\right)^n-ne\right)=?\]


7
Petrovic Eşitsizliğinin kullanıldığı bir Crux Mathematicorum sorusu paylaşayım..

Crux 3934.
$a,b,c$  herhangi bir üçgenin kenar uzunluklarını belirtmek üzere aşağıdaki eşitsizliği gösteriniz
$$\dfrac{a}{\sqrt[3]{4b^3+4c^3}}+\dfrac{b}{\sqrt[3]{4c^3+4a^3}}+\dfrac{c}{\sqrt[3]{4a^3+4b^3}}<2$$


8
Yakın zamanda, Daniel Sitaru tarafından ortaya atılmış bir Crux Mathematicorum problemini paylaşayım..

Crux 5005.
$x,y,z>0$  reel sayıları $xyz=1$  koşulunu sağlıyor. Aşağıdaki eşitsizliğin doğru olduğunu gösteriniz.
$$\left(\dfrac{x}{1+x+xy}+\dfrac{y}{1+y+yz}+\dfrac{z}{1+z+zx}\right)^3\leq \dfrac{x^3}{1+x+xy}+\dfrac{y^3}{1+y+yz}+\dfrac{z^3}{1+z+zx}$$


9
Müsait olan arkadaşlar bakabilir mi?


10
(Hüseyin Emekçi): Her $a,b$  ve $c$  pozitif reel sayıları için
$$\sum_{cyc}{\dfrac{1}{b\left(a^4+a^3c+b^2c^2\right)}}\geq \dfrac{27}{(a+b+c)(a^2+b^2+c^2)^2}$$
olduğunu ispat ediniz.


11
Her $1\leq i\leq n$ için $\lambda_i$ pozitif, $a_i$ ise negatif olmayan reel sayılar (hepsi birden $0$'a eşit değil) olmak üzere $k\geq 2$ için


$$\sum_{cyc- j}{\dfrac{\sqrt[k-1]{\lambda_j^k}a_j+\sqrt[k-1]{\lambda_{j+1}^k}a_{j+1}+\cdots+\sqrt[k-1]{\lambda_{j-1}^k}a_{j-1}}{\lambda_ja_j+\lambda_{j+1}a_{j+1}+\cdots+\lambda_{j-1}a_{j-1}}}\geq \sqrt[k-1]{\sum\limits_{cyc}{\lambda_1}}$$


olduğunu gösteriniz.


12
Genelleştirme 1
Her $1\leq i\leq n$  için  $\dfrac{\left(n-1\right)\left(a_i+a_{i+1}\right)}{2}>a_{i+2}+a_{i+3}+\cdots+a_{i-1}>|a_i-a_{i+1}|$  koşulunu sağlayan  $a_1,a_2,\cdots,a_n$ ($n\geq 3$)  pozitif reelleri için


$$\sum_{cyc- j}{\left(a_j^2a_{j+1}\cdots a_{j-2}\left(\dfrac{\left(n-1\right)a_{j}}{2}-\left(a_{j+1}+a_{j+2}\cdots+a_{j-2}\right)\right)\right)}\geq 0$$


eşitsizliği doğrudur.


13
Genelleştirme 1
$a_1,a_2,\cdots,a_n$ ($n\geq 2$) pozitif reel sayıları
$$\sum_{cyc}{a_1^{n-1}}=\dfrac{1}{\left(n-1\right)^{n-1}}$$
eşitliğini sağlasınlar. Buna göre
$$\sum_{cyc}{\dfrac{1}{\sqrt{a_1^2+a_2^2+\cdots+a_{n-1}^2}}}\leq \dfrac{\sqrt{n}}{\prod\limits_{cyc}{\left(a_1+a_2+\cdots+a_{n-1}\right)}}$$
olduğunu gösteriniz.


14
Genelleştirme 1
Her $a_1,a_2,\cdots,a_n$ ($n\geq 2$) pozitif reelleri için


$$\sum_{cyc- j}{\dfrac{a_j^3}{a_j^3+a_{j+1}^3+a_ja_{j+1}\left(a_{j+2}+a_{j+3}+\cdots+a_{j-1}\right)}}\geq 1$$


olduğunu gösteriniz.


15
Bu bağlantıdaki yani aşağıdaki problemin genellenmiş halini #2 de görmektesiniz.

Problem (Romanya District #9.3)
$a+b+c+d=80$  olmak üzere
$$a+\frac{b}{1+a}+\frac{c}{1+a+b}+\frac{d}{1+a+b+c}=8.$$
eşitliğini sağlayan tüm $a,b,c$  ve $d$  pozitif reellerini belirleyiniz.


16
Soru: Analitik düzlemde, merkezi $(2, \frac{1}{2})$ noktasında olan $r$ yarıçaplı çember ile $xy=1$ hiperbolü $4$ farklı noktada kesişiyor. Bu noktalardan üç tanesi eşkenar üçgen oluşturduğuna göre $r$ kaçtır?


17
$0\leq a\leq b\leq c$  koşulunu sağlayan her $a,b,c$  için $a+b+c=1$  ise


$$ab\sqrt{b-a}+bc\sqrt{c-b}+ca\sqrt{c-a}<\dfrac{1}{4}$$


olduğunu gösteriniz.


18
$$\dfrac{4a+9b+25c}{2a+3b+5c}+\dfrac{4b+9c+25a}{2b+3c+5a}+\dfrac{4c+9a+25b}{2c+3a+5b}=10$$

eşitliğini sağlayan tüm $(a,b,c)$  negatif olmayan reel üçlülerini bulunuz.


19
Problem. Aşağıdaki trigonometrik denklemi sağlayan

$$\dfrac{\sin\left(60^{\circ}-\dfrac{\alpha}{2}\right)}{\sin\left(60^{\circ}+\dfrac{\alpha}{2}\right)}\cdot \dfrac{\sin(\alpha+30^{\circ})}{\sin \alpha}\cdot \dfrac{\sin(2\alpha)}{\sin(2\alpha+30^{\circ})}=1$$

tüm  $\alpha \in \left(0,\dfrac{\pi}{3}\right]$  değerlerini bulunuz.


20
Her $1\leq i\leq n$ için $a_i$ pozitif reel sayılar olmak üzere $k\geq 1$ için


$$\left(\dfrac{a_1}{a_2+a_3+\cdots+a_n}\right)^k+\left(\dfrac{a_2}{a_3+a_4+\cdots+a_1}\right)^k+\cdots+\left(\dfrac{a_n}{a_1+a_2+\cdots+a_{n-1}}\right)^k\geq \dfrac{n}{\left(n-1\right)^k}$$


olduğunu gösteriniz.


21
Herhangi bir üçgenin kenarları $a,b,c$ için
 $2S=a+b+c$  olmak üzere $n\in N$ ise


$$\dfrac{a^k}{b+c}+\dfrac{b^k}{c+a}+\dfrac{c^k}{a+b}\geq \left(\dfrac{2}{3}\right)^{k-2}.S^{k-1}$$


olduğunu gösteriniz.


22
Her $x,y$  reel sayıları için
$$f(xf(x-y))+yf(x)=x+y+f(x^2)$$
ifadesini sağlayan tüm $f: \mathbb{R} \rightarrow \mathbb{R}$  fonksiyonlarını belirleyiniz.


23
Junior Balkan 2023 Shortlist'indeki analiz sorularının çoğunu forumda paylaşıp ispatlamıştık. Shortlist'in A.3 Problemine de yer verelim:

JBMO Shortlist 2023 #A.3: Her $x$, $y$  ve $z$  pozitif reelleri için $xy+yz+x=1$  ise

$$\dfrac{2}{xyz}+9xyz\geq 7(x+y+z)$$

olduğunu gösteriniz.


24
Her $a,b,c,d$ pozitif reel sayıları için  $abcd=1$  ise


$$\sqrt{\dfrac{a}{b+c+d^2+a^3}}+\sqrt{\dfrac{b}{c+d+a^2+b^3}}+\sqrt{\dfrac{c}{d+a+b^2+c^3}}+\sqrt{\dfrac{d}{a+b+c^2+d^3}}\leq 2$$


olduğunu gösteriniz.



25
$a\geq b\geq 1\geq c\geq 0$  koşulunu sağlayan tüm $a,b,c$ reel sayıları için $a+b+c=3$  ise


$$3\left(\dfrac{a}{b}+\dfrac{b}{a}\right)\geq 4c^2+\dfrac{a^2}{b}+\dfrac{b^2}{a}$$


olduğunu gösteriniz.


26
Elemanlari düzlem üzerindeki noktalar olan sonlu bir küme düşünelim. Bu kümenin içinde birbirine en uzak iki noktanın arasındaki uzaklığa kümenin çapı diyelim. Kümenin içinden herhangi bir $P$ noktası seçelim. $P$ noktasına en uzak olan noktanın $P$ ye mesafesinin, kümenin çapının en az yarısı olduğunu kanıtlayın.


27
Herhangi $x$  gerçeli için aşağıdaki ifadenin
$$\sqrt{x^{2}-3x+9}+\sqrt{x^{2}-5\sqrt{3}x+25}$$
alabileceği minimum değeri belirleyiniz.


28
Her $a,b,c$ pozitif reel sayılari için


$$\dfrac{a^3}{a^3+b^3+abc}+\dfrac{b^3}{b^3+c^3+abc}+\dfrac{c^3}{c^3+a^3+abc}\geq 1$$


olduğunu gösteriniz.


29
$\mathcal{C}[a,b]$, $[a,b]$ aralığında sürekli fonksiyonların kümesi olsun. $\mathcal{C}[a,b]$'da iç çarpımı şu şekilde tanımlayalım, $$\langle f,g\rangle=\int_{a}^{b}f(x)g(x)dx.$$ $f_1,f_2,\dots,f_N\in\mathcal{C}[a,b]$ fonksiyonları $$\langle f_i,f_j\rangle=\delta_{i,j}=\begin{cases}1&i=j,\\ 0&i\neq j\end{cases}$$ şartını sağlıyorsa $f_1,f_2,\dots,f_N$ fonksiyonlarına orthonormal denir.

Soru: $p_1,p_2,\dots, p_N\in\mathcal{C}[a,b]$ fonksiyonları orthonormal olsun. $g(x,s)=\sum\limits_{n=1}^{N}p_n(x)p_n(s)$ olarak tanımlayalım. Bu durumda herhangi bir $y\in\mathcal{C}[a,b]$ fonksiyonu için $$f(x)=y(x)-\int_{a}^{b}g(x,s)y(s)ds$$ fonksiyonunu tanımlarsak, her $n=1,2,\dots,N$ için $\langle f,p_n\rangle=0$ olacağını gösteriniz.

Örnek olarak $[0,2]$ aralığı için $p_n(x)=\sin{(n\pi x)}$ olarak alabilirsiniz.


30
Tüm $a_1,a_2,\cdots,a_n$ pozitif reel sayıları için


$$\sum_{cyc- j}{\dfrac{a_j}{a_{j+1}^2+a_{j+2}^2+\cdots+a_{j-1}^2}}\geq \dfrac{4}{a_1+a_2+\cdots+a_n}$$


olduğunu gösteriniz.


31
Genelleştirme 1
Her $a_1,a_2,\cdots,a_n$ pozitif reel sayılar olmak üzere $p\geq 2$ tam sayısı için


$$\sum_{cyc- j}{\dfrac{a_j}{a_{j+1}^p+a_{j+2}^p+\cdots+a_{j-1}^p}}\geq \dfrac{p^p}{\left(\left(p-1\right).\sum\limits_{cyc}{a_1}\right)^{p-1}}$$


olduğunu gösteriniz.


32
Çok bilinen ama tahminimce çoğu kişinin ispatı üzerine düşünmediği bir teoreminin ispatını tartışalım istedim.

Teorem 1: $P$, reel katsayılı bir polinom olsun. Eğer $m,n\in\mathbb{R}$ olmak üzere $m+ni$ karmaşık sayısı $P$'nin bir kökü ise, eşleniği olan $m-ni$ de bir köküdür.

Teorem 2: $P$, rasyonel katsayılı bir polinom, $d$ ise tamkare olmayan bir pozitif tamsayı olsun. Eğer $m,n\in\mathbb{Q}$ olmak üzere $m+n\sqrt{d}$ sayısı $P$'nin bir kökü ise, eşleniği olan $m-n\sqrt{d}$ de bir köküdür.


33
Müsait olan arkadaşlar bakabilir mi?


34
Problem (USAMO 1983/2)
Aşağıdaki 5. dereceden denklemde
$$x^5+ax^4+bx^3+cx^2+dx+e=0$$
$2a^2>5b$  ise beş kökün hepsinin birden pozitif reel olamayacağını ispatlayınız.


35
Her $x,y$ pozitif reel sayıları için

$$f\left(xy+f\left(x\right)\right)=xf\left(y\right)+2$$

koşulunu sağlayan tüm $f: \mathbb{R^{+}}\to\mathbb{R^{+}}$ tanımlanmış $f$ fonksiyonlarını bulunuz.


36
Her $x,y,z$  pozitif reeli için


$$\left(xy+yz+zx\right)\left(\dfrac{1}{(x+y)^2}+\dfrac{1}{(y+z)^2}+\dfrac{1}{(z+x)^2}\right)\geq \dfrac{9}{4}$$


olduğunu gösteriniz.



Sitemap 1 2 3 4 5 6 7 8 9 10 11 12 13 14 15 16 17 18 19 20 21 22 23 24 25 26 27 28 29 30 31 32 33 34 35 36 37 
SimplePortal 2.3.3 © 2008-2010, SimplePortal